Sample Exam 2

Pataasin ang iyong marka sa homework at exams ngayon gamit ang Quizwiz!

Zachary spent $7,000 on day-care services for his two children so that he could work. Assuming his earned and adjusted gross income is $50,000, how much is his dependent care credit? (Please ignore changes from the American Rescue Plan Act of 2021.) A) $1,200 B) $1,400 C) $7,000 D) $6,000

A) $1,200 The correct answer is (A).The dependent care credit is never completely phased out, and it provides a credit of 20 percent on up to $3,000 per qualifying child with a maximum of $6,000 for two or more children if the taxpayer has an AGI exceeding $43,000. Therefore, Zachary's dependent care credit is calculated as $6,000 × 0.20 = $1,200. LO 9.2

Gwen purchased 60 shares of Banana Inc., for $9,000 6 months ago. Unfortunately, the value of the shares has dropped to $6,000. Gwen's son, Gerrard, is heading off to college, and Gwen is tired of waiting for the stock to rise again in value. Gwen gives the stock to Gerrard when it is worth $6,000 to help fund his education. If Gerrard sells the shares 8 months after the transfer for $4,000, what is the amount and character of his gain or loss? A) $2,000 short-term capital loss B) $2,000 long-term capital loss C) $5,000 long-term capital loss D) $5,000 short-term capital loss

A) $2,000 short-term capital loss The correct answer is (A).Gwen gave loss property to her son. When a taxpayer transfers loss property and the recipient subsequently sells the property for an amount between the donor's adjusted basis and the loss basis defined as the fair market value of the stock on the date of the gift, no gain or loss is recognized. In this example, the "no gain/no loss" corridor ranges from $6,000 (the fair market value of the stock on the date of the gift) to $9,000 (the adjusted basis of the donor). If Gerrard sells the stock for any amount between $6,000 and $9,000, he does not recognize any gain OR loss on the transaction. In this example, $6,000 is used to determine the loss. Because this is a loss transaction, Gerrard's handling period is 8 months classifying the sale as a short-term capital loss. Gwen's holding period of 6 months at the time of the gift is disregarded. Long story short...Gerrard's basis in the gifted stock was $6,000. He sold it for $4,000 (8 months after Gwen gave him the stock) resulting in a short-term capital loss of $2,000. LO 11.1

Ethan, an unmarried taxpayer using the single filing status, received $14,000 of Social Security retirement benefits this year. Ethan also received $9,000 of interest income and $19,000 of income from his retirement plan during the year. How much of Ethan's Social Security benefits must be included in his gross income this year? A) $5,350 B) $0 C) $7,000 D) $11,900

A) $5,350 The correct answer is (A).Since his modified adjusted gross income (MAGI) of $28,000 plus one-half of his Social Security benefits (0.5 × $14,000 = $7,000) exceeds his adjusted base amount of $34,000, he must calculate his includible Social Security benefits using the lesser of the amount calculated using formula 3 or formula 4: Formula 3: 0.85 × $14,000 = $11,900 Formula 4: 0.85 × [$28,000 + (0.50 × $14,000) − $34,000] = $850 plus the least of $4,500, the amount calculated using formula 1, and the amount calculated using formula 2: Formula 1: 0.50 × $14,000 = $7,000 Formula 2: 0.50 × [$28,000 + (0.50 × $14,000) − $25,000] = $5,000 $4,500 is the least of these three numbers. The formula 4 total is $5,350, or $850 + $4,500. The lesser of the amounts for formula 3 and formula 4 is $5,350. Therefore, $5,350 of the Social Security benefits must be included in Ethan's gross income. LO 5.6

Aaron is covered by a group term life insurance policy that provides coverage equal to 2.5 times his annual salary of $300,000. Aaron's employer pays for 60% of the premiums of the policy, for which the uniform premium is $0.15 per $1,000 of coverage per month. How much of this premium is taxable to Aaron? A) $756 B) $1,350 C) $1,260 D) $810

A) $756 The correct answer is (A).The cost of up to $50,000 of coverage is nontaxable. $750,000 − $50,000 = $700,000 Only the 60% of the value of the premium paid by the employer would be taxable to Aaron. [($700,000 ÷ $1,000) × $0.15] × 12 months × 60% = $756 taxable coverage LO 5.3

On January 1, 2021, Wyatt and Wendy divorced. Under the terms of the divorce decree, Wendy was given custody of their only child, Autumn, who is 14 years old. The court decree also requires Wyatt to pay Wendy $2,000 a month in alimony for 4 years (that is, until Autumn turns 18), followed by $800 a month for the 4 years after that. Which of the following statements concerning the payments that Wyatt makes to Wendy is correct? A) During the first 4 years, $800 and $1,200 of the monthly payments would be classified as alimony and child support, respectively. B) Wendy will need to include both the alimony and child support payments in gross income. C) During the first 4 years, the monthly payments of $2,000 would be classified as child support. D) During the first 4 years, the monthly payments of $2,000 would be classified as alimony.

A) During the first 4 years, $800 and $1,200 of the monthly payments would be classified as alimony and child support, respectively. The correct answer is (A).For income tax purposes, alimony payments that are reduced when a minor child reaches the age of majority (18 in most states) are considered to be a form of child support. Since Wyatt's alimony payments will be reduced by $1,200 per month once Autumn reaches age 18, $800 of the payment will be considered alimony, leaving the remaining $1,200 as child support. LO 6.5

Several years ago, Francis purchased a piece of equipment for $3,000 and has since taken $1,000 in depreciation deductions. Francis is now ready to replace the equipment with a new item, but he does not know how to calculate the tax consequences of selling the equipment. Which of the following statements is true regarding the tax consequences of selling the old piece of equipment? A) If Francis sells the old weight machine for $1,400, he will have a $600 ordinary loss. B) If Francis sells the old weight machine for $2,200, he will have a $200 capital gain. C) If Francis sells the old weight machine for $2,000, he will have a $1,000 capital gain. D) If Francis sells the old weight machine for $3,500, he will have a $1,500 capital gain.

A) If Francis sells the old weight machine for $1,400, he will have a $600 ordinary loss. LO 12.2

Shawn is a 5 percent owner in Logger Pro, LLC. He does not materially participate in the business. His at-risk amount in the company is $200,000, and his share of the loss generated by the company for the year is $230,000. Assuming there is no other passive income generated, which of the following statements is true? A) The loss suspended because of the at-risk rules is $30,000, and the loss suspended because of the passive-activity loss rules is $200,000. B) The loss suspended because of the at-risk rules is $230,000, and the loss suspended because of the passive-activity loss rules is $0. C) The loss suspended because of the at-risk rules is $200,000, and the loss suspended because of the passive-activity loss rules is $30,000. D) The loss suspended because of the at-risk rules is $0, and the loss suspended because of the passive-activity loss rules is $230,000.

A) The loss suspended because of the at-risk rules is $30,000, and the loss suspended because of the passive-activity loss rules is $200,000. The correct answer is (A).Of the $230,000 loss from Logger Pro, LLC, $30,000 will be suspended because of the at-risk rules (he only has $200,000 at risk) and $200,000 will be suspended because of the passive-activity loss rules. LO 14.2

GME, Inc., has a net income of $300,000 before depreciation and has purchased one used piece of equipment during the year for $400,000. Which of the following depreciation strategies will allow GME to claim the largest depreciation deduction in the current year? A) The use of Section 179 B) The use of the modified accelerated cost recovery system (MACRS) C) The use of straight-line depreciation D) The use of 50% bonus depreciation

A) The use of Section 179 The correct answer is (A).Fifty percent bonus depreciation allows a 50% write-off against income plus regular depreciation under MACRS, whereas Section 179 allows the taxpayer to deduct the lesser of $1,050,000 (for 2021) and the net income of $300,000. While MACRS allows depreciation to be claimed on an accelerated basis, it will take 5 years to recover the full cost of the equipment according to the depreciation tables. LO 10.3

In order to keep her full-time job, Michaela spent $5,000 on day-care services for her three children. Assuming her earned and adjusted gross income is $15,000, how much is her dependent care credit? (Please ignore changes from the American Rescue Plan Act of 2021.) A) $5,000 B) $1,750 C) $6,000 D) $1,000

B) $1,750 The correct answer is (B).Since Michaela's AGI is $15,000 or less, she is entitled to claim a dependent care credit equal to 35 percent on up to $3,000 per qualifying child with a maximum of $6,000 for two or more children. Therefore, her dependent care credit is calculated as $5,000 × 0.35 = $1,750. LO 9.2

Jefferson has a real estate asset used in his business. The asset is worth $80,000. He exchanges it for a like-kind real estate asset owned by Teddy, which is worth $49,000. The basis of Jefferson's asset is $70,000. In order to make the exchange fair, Teddy contributes $31,000 cash plus the asset in exchange for Jefferson's asset. Teddy's basis in his original asset is $35,000. What is Jefferson's gain or loss? A) $14,000 gain realized B) $10,000 gain realized and recognized C) $0 gain recognized D) $0 loss recognized

B) $10,000 gain realized and recognized LO 13.1

Archie and Amanda got married and bought a house 9 months ago for $600,000. Archie's job recently transferred him to an office in a different state, so the couple was forced to sell their house. What amount of gain from the sale of the personal residence must Archie and Amanda include in their gross income if the house sells for $900,000, assuming they use the MFJ tax status? A) $187,500 B) $112,500 C) $300,000 D) $0

B) $112,500 The correct answer is (B).Although they did not live in their house for a full 2 years, Archie and Amanda are eligible for a prorated exclusion because of Archie's change in employment. Therefore, they are eligible for a maximum exclusion of $187,500, that is, 9⁄₂₄ × $500,000, and must include $112,500 in gross income as a result. LO 13.4

Jason, aged 15 and a qualifying dependent of his parents, has $3,500 in interest and dividends and $12,000 in earned income from a part-time job. What is Jason's standard deduction this year? A) $15,500 B) $12,400 C) $12,550 D) $12,000

B) $12,400 The correct answer is (B).The standard deduction for a qualifying dependent is the lesser of earned income plus $400 or the standard deduction of $12,950 in 2022. LO 3.5

Angus is a 40 percent partner in a partnership that produced business income (before adjustments) of $100,000 this year, and he is a 100 percent owner in an S corporation that produced net business income of $90,000 this year. The deduction for self-employment taxes allocated to his net business income is $2,826. His total taxable income is $125,000. What are Angus's qualified business income (QBI) and QBI deduction, respectively? A) $127,175 and $25,435 B) $127,174 and $25,000 C) $125,000 and $25,000 D) $125,000 and $22,435

B) $127,174 and $25,000 The correct answer is (B).Angus's QBI from the partnership is $37,174, that is, 40% × $100,000 − $2,826, and his QBI from the S corporation is $90,000. His total QBI is $127,174, that is, $37,174 + $90,000. His QBI deduction is $25,000, that is, 20% × $125,000 since the taxable income amount is lower than his QBI. LO 16.3

Tucker is single, and his share of qualified business income (QBI) from a partnership is $105,000. His AGI is $130,000, which includes $10,000 of capital gains income and $5,000 of self-employment taxes that Tucker paid. Assuming Tucker does not itemize, what is the amount of the Section 199A qualified business income deduction that he can claim this year? A) $21,000 B) $20,490 C) $23,490 D) $20,400

B) $20,490 The correct answer is (B).The deduction is the lesser of 20 percent of QBI and 20 percent of taxable income (less net capital gains). To arrive at taxable income, we must first reduce his AGI by the standard deduction. $130,000 − $12,550 = $117,450. We then must reduce this amount by the $15,000 of capital gains and self-employment taxes to arrive at $102,450. Twenty percent of his QBI is $21,000. Twenty percent of his taxable income (less net capital gains) is $20,490. He is entitled to claim a QBI deduction in the amount of $20,490. LO 7.5

The McGill Company granted Larry 1,000 incentive stock options (ISOs) on January 5 of Year 1. The exercise price is $20. The market price on the exercise date (July 15 of Year 2) is $45. What is the alternative minimum tax (AMT) consequence when Larry exercises the ISO? A) $0 AMT gain per share B) $25 AMT gain per share C) $20 AMT gain per share D) $45 AMT gain per share

B) $25 AMT gain per share The correct answer is (B).The AMT gain per share is the difference between the market price and the exercise price at the date of exercise. $45 − $20 = $25 LO 15.2

On January 1 of this year, Patti purchased a home worth $2.5 million with an interest-only mortgage of $1.6 million. She is currently only paying interest on the mortgage at the rate of 3 percent annually. What amount of qualified residence interest is not deductible as home mortgage interest on Schedule A of her individual income tax return this year? A) $22,500 B) $25,500 C) $48,000 D) $27,000

B) $25,500 The correct answer is (B).Since Patti purchased her home after December 15, 2017, the deduction for qualified residence interest is limited to the interest on $750,000 of principal. Any interest paid on a loan in excess of this amount is not deductible. $1.6 million × 3% = $48,000 $750,000 × 3% = $22,500 $48,000 − $22,500 = $25,500 nondeductible interest LO 7.2

Amelia purchased series EE savings bonds for $10,000 at the age of 36. This year she redeemed the bonds for $12,000 and paid qualified higher education expenses in the amount of $9,600 for her son. How much interest must Amelia include in her gross income this year? A) $0 B) $400 C) $1,600 D) $2,000

B) $400 The correct answer is (B).Because Amelia did not use all the proceeds from the bond redemption to pay for qualified education expenses, she will be required to include part of the interest income from the bonds in her gross income. Amelia may exclude $1,600 of interest income from her gross income, that is, ($9,600 ÷ $12,000) × $2,000. Therefore, Amelia must include $400 in her gross income, or $2,000 − $1,600. LO 4.4

Florence and Greyson, both aged 51, are married and filed a joint federal income tax return for 2021. Florence earned a salary of $160,000 and was covered by her employer's pension plan. Greyson earned a salary of $20,000 and was not covered by his employer's pension plan. What is the maximum amount of traditional IRA contributions they can deduct in 2021? A) $0 B) $7,000 C) $6,000 D) $14,000

B) $7,000 The correct answer is (B).The phaseout for married taxpayers filing jointly begins at $204,000 and ends at $214,000 (in tax year 2022) for the spouse who is not covered by a workplace retirement plan but whose spouse is covered. Therefore, Florence would not be able to make a deductible IRA contribution, but Greyson would be able to make a full deductible contribution in the amount of $7,000 since Florence's Modified AGI is less than the $204,000 threshold. LO 6.4

Assume a taxpayer incurred business losses in Year 1 and generated an NOL of $28,000 that would be carried forward to offset income in future years. Assuming the taxpayer had $34,000 of taxable income and incurred no additional business income or loss in Year 2, what amount of NOLs would be carried forward to Year 3? A) $0 B) $800 C) $27,200 D) $5,600

B) $800 The correct answer is (B).NOLs can only be carried forward and are limited to 80 percent of taxable income. 80 percent × $34,000 = $27,200 $27,200 of the $28,000 NOL carryforward may be recognized in Year 2, leaving the remainder ($800) to be carried forward to Year 3. LO 8.3

Nadia is a single taxpayer who had the following items of income and expense for the current tax year: Wages: $85,000 U.S. savings bond interest: $5,000 Ordinary dividends: $3,000 Municipal bond interest: $2,500 Receipt of gift from brother: $75,000 Federal taxes paid: $9,000 Real estate taxes paid: $7,000 Mortgage interest paid: $8,000 What is Nadia's adjusted gross income (AGI) for the current year? A) $95,500 B) $93,000 C) $146,500 D) $84,000

B) $93,000 The correct answer is (B).Nadia's income includes the wages, U.S. savings bond interest, and ordinary dividends. Therefore, her AGI is $93,000. $85,000 + $5,000 + $3,000 = $93,000 LO 3.2

Pamela passed away 3 months after she purchased a vacation condo in Orlando for $60,000. The fair market value of the property as of the date of Pamela's death was $55,000. She bequeathed the property to her son, Parker. Since Parker was the only beneficiary of the estate and there were no estate taxes due, the title to the property was transferred within one month of Pamela's death. Six months after receiving title to the property, Parker sold the property for $60,000. What is the amount and type of income that Parker will report on the sale? A) A $5,000 short-term capital gain B) A $5,000 long-term capital gain C) A $5,000 short-term capital loss D) No gain or loss will be reported on the sale.

B) A $5,000 long-term capital gain The correct answer is (B).When Pamela died, the basis of the Orlando property qualified for a step to fair market value under IRC Section 1014. Parker's basis in the property is therefore $55,000. Since he sold the property for $60,000 and his basis was $55,000, Parker's gain is $5,000. Even though Pamela purchased the property 10 months before it was sold, Parker's gain will be treated as a long-term capital gain. Any property received through the estate of a decedent automatically qualifies for long-term capital-gains treatment. LO 11.1

Keith owns a downtown office building. He originally purchased the building for $600,000 and took straight-line depreciation deductions of $400,000. What will the tax consequences be if Keith sells the building for $500,000? (Ignore the net investment income tax.) A) Keith will have a $300,000 gain taxed at long-term capital-gains rates. B) Keith will have a $300,000 gain taxed at a rate of 25 percent. C) Keith will have a $300,000 gain taxed at ordinary income tax rates D) Keith will not recognize any gain or loss on the sale of the building.

B) Keith will have a $300,000 gain taxed at a rate of 25 percent. The correct answer is (B). Sale Price$500,000 Less Adjusted Basis (that is, $600,000 − $400,000)− $200,000 Equals Gain$300,000 Breakdown of Gain: Keith will have a $300,000 unrecaptured Section 1250 gain (straight-line depreciation) taxed at 25%. LO 12.2

Taylor has a 15 percent interest in a general partnership for which she invested $60,000 and does not materially participate in the operations of the business. Assume the total partnership loss for the year is $450,000. How much can Taylor claim as a loss in the current year, assuming she has $100,000 of passive income from other sources and her partnership basis is equal to her original investment? A) $0 B) $67,500 C) $60,000 D) $100,000

C) $60,000 The correct answer is (C).Taylor's loss is limited to $60,000 due to both the basis and at-risk rule limitations. LO 14.2

Assume a taxpayer incurred business losses in Year 1 and generated an NOL of $15,000 that would be carried forward to offset income in future years. Assuming the taxpayer had $20,000 of taxable income and incurred no additional business income or loss in Year 2, what amount of NOLs could be recognized in Year 2? A) $20,000 B) $16,000 C) $15,000 D) $12,000

C) $15,000 The correct answer is (C).NOLs can only be carried forward and are limited to 80 percent of taxable income. In this case, 80 percent of Year 2 taxable income ($20,000) is equal to $16,000 and exceeds the amount of the NOL carryforward from Year 1, thus the entire NOL carryforward of $15,000 can be recognized and will reduce gross income in Year 2. LO 8.3

In Year 8, Alan had a Section 1231 gain of $20,000. In the prior years, Alan had the following Section 1231 transactions: YearNet Section 1231 Transaction Year 7$9,000 Section 1231 Loss Year 6$2,000 Section 1231 Loss Year 5$4,000 Section 1231 Loss Year 4No Section 1231 Transaction Year 3$10,000 Section 1231 Gain Year 2$8,000 Section 1231 Loss Year 1$5,000 Section 1231 Loss How will Alan's Section 1231 gain be taxed in Year 8? A) $20,000 will be taxed as ordinary income. B) $20,000 will be taxed as a Section 1231 capital gain. C) $15,000 will be taxed as ordinary income, and $5,000 will be taxed as a Section 1231 capital gain. D) $5,000 will be taxed as ordinary income, and $15,000 will be taxed as a Section 1231 capital gain.

C) $15,000 will be taxed as ordinary income, and $5,000 will be taxed as a Section 1231 capital gain. The correct answer is (C). 5-Year Look-Back Rule Year Net Section 1231 Transaction Year 7$9,000 Section 1231 Loss Year 6$2,000 Section 1231 Loss Year 5$4,000 Section 1231 Loss Year 4No Section 1231 Transaction Year 3$10,000 Section 1231 Gain Year 2$8,000 Section 1231 Loss Year 1$5,000 Section 1231 Loss Alan would have to recognize $15,000 of his 2021 gain as ordinary income since in 2020, 2019, and 2018 he had Section 1231 losses. The Section 1231 5-year look-back rule requires that a net Section 1231 gain in the current tax year will be taxed at ordinary income tax rates to the extent of any unrecaptured Section 1231 losses claimed during the last 5 years. The remaining $5,000 of Alan's Section 1231 gain (that is, $20,000 − $15,000) would be treated as a Section 1231 capital gain. Just so you know, the $10,000 gain in Year 3 has no effect on the 5 year look-back on the Year 8 income. It was a gain, not a loss! LO 12.3

Don and Betty are married, use the MFJ tax status, and never itemize their deductions. They have a gross income of $250,000 and deductions for adjusted gross income (AGI) in the amount of $12,000. Don and Betty are both over the age of 65, and neither is blind. What is Don and Betty's taxable income in the current year? Assume a standard deduction of $25,900 and additional standard deduction of $1,400. A) $212,900 B) $224,900 C) $209,300 D) $211,550

C) $209,300 The correct answer is (C).Don and Betty's taxable income = gross income − deductions for adjusted gross income − the greater of the standard deduction or itemized deductions + additional deductions for being over the age of 65. Therefore, their taxable income can be calculated as follows: For the Tax Year 2021 Gross Income$250,000 Less: Deductions for AGI− $12,000 Adjusted Gross Income$238,000 Less: Standard Deduction−$25,900 Less: Additional Standard Deduction− $2,800 Taxable Income$209,300 LO 3.3

Francesca, an unmarried taxpayer and aged 67, still works as an employee for the law offices of Wexler McGill. Her only sources of income this year are $200,000 of W-2 wages, $25,000 in Social Security benefits, and $7,500 in U.S. savings bond interest. Based on this information, Francesca's adjusted gross income (AGI) for the current year is A) $221,250. B) $207,500. C) $228,750. D) $232,500.

C) $228,750. The correct answer is (C).Francesca's adjusted gross income can be calculated as follows: W-2 Income$200,000 U.S. Savings Bond Interest$7,500 Social Security Benefits (85% × $25,000)+ 21,250 $228,750 Her Social Security benefits are 85 percent taxable because of her modified AGI. LO 5.6

Helen invested $30,000 in an annuity contract many years ago. This year, Helen annuitized the contract. The insurance company agreed to pay Helen $222.22 per month for 15 years. If Helen receives five payments this year, how much must she include in her gross income this year? A) $0 B) $833.33 C) $277.77 D) $1,111.10

C) $277.77 The correct answer is (C).Helen's expected return is about $40,000, that is, 15 years × 12 months × $222.22. Therefore, her exclusion ratio is 75 percent, or $30,000 ÷ $40,000. Helen will receive $1,111.10 in annuity payments this year (five payments × $222.22), of which $833.33 can be excluded. Therefore, Helen must include $277.77, or 25% × $222.22 × 5, in her income this year. LO 4.3

Julian owns a rental home in Oregon. He decided that he would like to acquire a rental home in Maine. Alice, who lives in Maine, has a rental home there. Julian and Alice decide to exchange properties via a qualified intermediary. Julian's basis in his property is $400,000, and Alice's basis in her property is $525,000. Julian and Alice exchange the two properties. However, Julian needs to give Alice an additional $110,000 in cash since the fair market value of his property is $450,000, while the fair market value of her property is $560,000. What is Alice's recognized gain or loss as a result of the transfer? A) No gain or loss recognized in a 1031 exchange B) $110,000 gain C) $35,000 gain D) $35,000 loss

C) $35,000 gain LO 13.1

Megan and Mason are married taxpayers who use the MFJ tax status and have one dependent child, aged 19. The couple had W-2 income of $300,000. $59,000 in federal taxes was withheld, and the couple incurred the following expenses during the year: Interest on a home equity loan issued for $100,000 and used to purchase a second vehicle$3,000 Interest on their first mortgage with an original loan balance of $600,000$8,000 Property taxes on a personal residence$7,500 State income tax$10,000 Qualifying charitable contributions$5,000 Which of the following best describes their final tax outcome? (Assume a flat tax rate of 24 percent and that their standard deduction is $25,900.) (Please ignore changes from the American Rescue Plan Act of 2021.) A) $7,480 tax balance due B) $6,976 tax balance due C) $6,284 tax balance due D) $6,980 tax balance due

C) $6,284 tax balance due The correct answer is (C).Their refund can be determined by subtracting the applicable credits and their withholdings from their tax liability. They are eligible for a family tax credit for the 19-year-old. $300,000 of income minus the standard deduction of $25,900 results in $274,100. Taxed at a rate of 24 percent, their tax liability is $65,976. 0.24 × $274,100 = $65,784 Family credit: $500 (one child over age 16) Taxes withheld: $59,000 $65,784 − $500 − $59,000 = $6,284 Megan and Mason have a tax balance due of $6,284. LO 9.1

In Year 1, Ben and Nicole have an alternative minimum taxable income (AMTI) of $1,210,000. Assuming they use the MFJ tax status and the AMT exemption phaseout starts at $1,047,200 and their AMT exemption is $114,600, what amount of the alternative minimum tax (AMT) exemption are Ben and Nicole eligible to use this year? A) $40,700 B) $114,600 C) $73,900 D) $162,800

C) $73,900 The correct answer is (C).Because their AMTI is above the AMT phaseout threshold amount, their AMT exemption must be reduced. Ben and Nicole's exemption is reduced by 25 percent of the amount that their AMTI exceeds $1,047,200 (the threshold). Therefore, their exemption must be reduced by $40,700, that is, ($1,210,000 − $1,047,200) × 0.25. As a result, Ben and Nicole are entitled to an exemption of $73,900, that is, $114,600 − $40,700. LO 15.1

AMC, Inc., has a net income of $2,800,000 before depreciation and has purchased two pieces of equipment during the year. The first piece of equipment cost $2,000,000, and the second piece of equipment cost $900,000. How much may be expensed under Section 179 for the current year for both pieces of equipment? A) $280,000 B) $1,050,000 C) $880,000 D) $2,900,000

C) $880,000 The correct answer is (C).The Sec. 179 expensing maximum is $1,080,000 (for 2022). This amount is reduced dollar-for-dollar by the amount of depreciable property placed in service during the year exceeding $2,700,000 (for 2022). The total placed in service was $2,900,000, which exceeds $2,700,000 by $200,000. The reduced limit is therefore $1,080,000 − $200,000 = $880,000. LO 10.4

Otis would like to invest in bonds and is considering either a corporate taxable bond with an interest rate of 9 percent or a tax-exempt municipal bond of comparable risk and quality. Assuming Otis's marginal tax rate is 24 percent, what interest rate must the municipal bond earn to equal the after-tax return than the corporate bond? A) 6.50% B) 6.62% C) 6.84% D) 6.91%

C) 6.84% The correct answer is (C). The equivalent tax-free rate for the taxable bond is 6.84 percent. Therefore, the municipal bond must earn at least this amount to exceed the return provided by the corporate bond on an after-tax basis. 0.09 × (1 − 0.24) = 6.84% LO 4.4

Olivia has been adjusting her investment portfolio to meet her target asset allocation and has realized the following capital gains and losses this year: $27,000 in short-term capital gains $25,000 in short-term capital losses $6,000 in long-term capital gains 13,000 in long-term capital losses What is Olivia's net capital gain or loss? A) A $2,000 net short-term capital gain (NSTCG) B) A $7,000 net long-term capital loss (NLTCL) C) A $5,000 net long-term capital loss (NLTCL) D) A $5,000 net short-term capital loss (NSTCL)

C) A $5,000 net long-term capital loss (NLTCL) The correct answer is (C).Olivia would have a $5,000 net long-term capital loss (NSTCL). LO 11.2

Holden purchased 75 shares of Mindhunter Enterprises for $1,000 per share 3 years ago. On March 1 of this year, Holden decided to sell 50 shares for a total of $48,000. On March 17 of the same year, Holden reads a market summary report that indicates the price of Mindhunter Enterprises is expected to increase substantially. As a result, he buys back 50 shares for $53,000. What are the tax consequences for Holden this year? A) Holden will have a $5,000 realized and recognized gain. B) Holden will have a $5,000 realized and recognized loss. C) Holden will have a $2,000 realized but not recognized loss. D) Holden will have a $2,000 realized and recognized loss.

C) Holden will have a $2,000 realized but not recognized loss. The correct answer is (C).Since Holden purchased and sold substantially identical securities within 30 days, a wash sale occurs. His realized loss on the sale of the original shares is calculated as follows: Amount Realized$48,000 Less: Adjusted Basis− $50,000 Equals: Gain or Loss-$2,000 Due to the wash sale transaction, Holden will not be permitted to recognize the loss in the year it was incurred. Instead, the realized but unrecognized loss of $2,000 will be added to the basis of the replacement securities. LO 11.3

Martin owns a condo in Scottsdale and an apartment in Manhattan. He spends most of his time in Scottsdale, so he sometimes rents out the apartment in Manhattan when he is not there. This year, Martin rented out the apartment for 150 days and personally used the apartment for 12 days. How will his rental activity be classified for tax purposes, and why? A) It will be classified as a nontaxable activity because Martin used the apartment personally more than he rented it out. B) It will be classified as mixed-use activity because Martin both rented out the apartment and used it personally. C) It will be classified as rental activity because Martin rented out the apartment for more than 14 days or 10% of the days the apartment was rented. D) It will be classified as mixed-use activity because Martin rented out the apartment for more than 14 days and personally used the apartment for the greater of 10 days and 10 percent of the rental days.

C) It will be classified as rental activity because Martin rented out the apartment for more than 14 days or 10% of the days the apartment was rented. The correct answer is (C).The rental of the Manhattan apartment will be classified as rental activity because the apartment was rented out for more than 14 days and the personal use did not exceed either 14 days or 10 percent of the rental days (15 days). LO 8.2

Victor owns a downtown office building. Victor originally purchased the building for $3,000,000, and he took depreciation deductions of $1,200,000. The straight-line depreciation would have been $900,000. What would the tax consequences be if Victor sold the building for $4,250,000? A) Victor would have ordinary income of $900,000. B) Victor would have $300,000 of unrecaptured Section 1250 gain. C) Victor would have capital gains of $1,250,000. D) Victor would have capital gains of $2,450,000.

C) Victor would have capital gains of $1,250,000. The correct answer is (C). Sale Price$4,250,000 Less Adjusted Basis (that is, $900,000 − $400,000)− $1,800,000 Equals Gain$2,450,000 Tax Treatment of Gain: LO 12.2

During the current year, a taxpayer collected $150 interest on U.S. Treasury bills, $850 interest on a local municipal bond, and $200 from a state income tax refund. (He did not itemize his deductions last year.) He also received $1,000 in dividends from a U.S. common stock. His gross income from the above is A) $1,200. B) $1,350. C) $2,000. D) $1,150.

D) $1,150. The correct answer is (D).Interest on the municipal bond is excluded under Section 103, and per the tax benefit rule, none of the $200 state tax refund should be included in gross income. His gross income is $1,150, or $150 + $1,000. If the taxpayer had instead itemized and received a benefit from deducting state income tax payments, he would have included the state income tax refund in gross income for the current year. LO 4.2

Riley, aged 16 and a qualifying dependent of her parents, has $12,650 of earned income from a newspaper route and $1,500 of unearned income. How much is taxable to Riley at her own tax rate? A) $500 B) $1,450 C) $1,150 D) $1,200

D) $1,200 The correct answer is (D).Since Riley's unearned income is below $2,200, all of her earned AND unearned income in excess of the standard deduction is taxed at her rate.Her standard deduction is limited to $12,950 (the standard deduction for a single person) since her earned income of $12,650 + $400 exceeds the standard deduction amount.Her total income is $14,150 − $12,950 standard deduction = $1,200. LO 3.6

Cora, an unmarried individual who is an executive at GEM Consulting, Inc., was granted 2,500 ISOs 2 years ago when the price per share was $8. The last few years have resulted in tremendous growth for GEM, and the stock is now trading at $90 per share. Cora exercised the ISOs but did not sell the stock; she plans on holding the shares for at least a year so she can pay the lower capital-gains tax rate on the growth. How much will Cora need to add to her taxable income when computing alternative minimum taxable income (AMTI) as a result of this transaction? A) $20,000 B) $225,000 C) $245,000 D) $205,000

D) $205,000 The correct answer is (D).Even though exercise of the options results in no taxable event for regular tax purposes this year, Cora will need to add $205,000, that is, ($90 − $8) × 2,500, to her taxable income when computing AMTI. If there are no other transactions this year that could reduce AMTI, it is likely that Cora will become an alternative minimum tax (AMT) taxpayer for the year since the tax preference item—the gain on the exercise of the ISO—is greater than her exemption for AMT purposes. LO 15.2

Stephanie is a 10 percent partner in a partnership that produced business income (before adjustments) of $300,000 this year, and she is a 100 percent owner in an S corporation that produced net business income of $20,000 this year. In addition, she received a salary of $75,000 from the S corporation. The deduction for self-employment taxes allocated to her net business income is $2,119. Her total taxable income is $80,000. What are Stephanie's qualified business income (QBI) and QBI deduction, respectively? A) $47,881 and $16,000 B) $122,881 and $16,000 C) $122,881 and $24,576 D) $47,881 and $9,576

D) $47,881 and $9,576 The correct answer is (D).Stephanie's QBI from the partnership is 27,881, that is, 10% × $300,000 − $2,119, and her QBI from the S corporation is $20,000, since wages are not considered part of QBI. Her total QBI is $47,881, that is, $27,881 + $20,000. Her QBI deduction is $9,576, that is, 20% × $47,881. LO 16.3

Mike and Laura got married and bought a house 16 months ago. Mike's job recently transferred him to an office in a different state, so Mike and Laura sold their house. If they originally purchased their home for $800,000, made $200,000 of improvements to the home, and sold it for $1,400,000, what is the amount of gain from the sale of the personal residence they must include in gross income? A) $0 B) $333,333 C) $133,333 D) $66,667

D) $66,667 The correct answer is (D).Although they did not live in their house for a full 2 years, Mike and Laura are eligible for a prorated exclusion because of Mike's change in employment. Therefore, they are eligible for a maximum exclusion of $333,333, that is, 16/24 × $500,000, and must include $66,667 of the $400,000 gain in gross income as a result. LO 13.4

Luisa is an 18 percent owner in Blue Fox, LLC. She is also a 12 percent owner in Red Box, LLC. She does not materially participate in either business. Her at-risk amount in Blue Fox is $150,000, and her share of income generated by the company this year is $60,000. For Red Fox, her at-risk amount is $100,000 and her share of loss generated by the company this year is $185,000. How much of Luisa's loss is suspended under the at-risk rules and passive-activity loss rules, respectively? A) $85,000 and $0 B) $40,000 and $85,000 C) $185,000 and $0 D) $85,000 and $40,000

D) $85,000 and $40,000 The correct answer is (D).Of the $185,000 loss from Red Fox, $85,000 will be suspended because of the at-risk rules (Luisa only has $100,000 at risk) and $40,000 will be suspended because of the passive-activity loss rules. (She only has $60,000 of passive income to offset the remaining loss). LO 14.2

Timothy is starting a new business. His top priority is limited liability, but he would also like to have flow-through taxation. At some point, he would like to be able to easily sell ownership interests in the business, and he expects to have more than 100 investors. Which of the following entities would best suit Timothy's needs? A) A general partnership B) An S corporation C) A C corporation D) An LLC

D) An LLC The correct answer is (D).Option (A) is incorrect because a general partnership does not provide limited liability. Option (C) is incorrect because a C corporation would not provide flow-through taxation. Option (B) is incorrect because an S corporation cannot take on more than 100 investors. Only Option (D), an LLC, meets all of Timothy's requirements. LO 16.1

Ava receives incentive stock options (ISOs) with an exercise price of $100 when the stock is trading at $100. Ava exercises these options 13 months after the date of the grant when the stock price is $125 per share. Which of the following statements is correct? A) Ava will have W-2 income of $25 per share upon exercise. B) Ava will have W-2 income of $100 per share when she is granted the options. C) Ava will have $100 of alternative minimum taxable income (AMTI) upon exercise. D) Ava adjusted basis for regular income tax will be $100 per share at exercise.

D) Ava adjusted basis for regular income tax will be $100 per share at exercise. The correct answer is (D).Ava's adjusted basis will be equal to $100 per share upon exercise. LO 5.3

Patrick purchased a piece of equipment for $175,000 and has since taken $80,000 in depreciation deductions. Which of the following statements is true regarding the tax consequences if Patrick decides to sell the equipment this year? A) If Patrick sells the equipment for $100,000, he will have a $5,000 capital loss. B) If Patrick sells the equipment for $50,000, he will have a $125,000 ordinary loss. C) If Patrick sells the equipment for $200,000, he will have a capital gain of $105,000. D) If Patrick sells the equipment for $120,000, he will have a $25,000 ordinary gain.

D) If Patrick sells the equipment for $120,000, he will have a $25,000 ordinary gain. LO 10.5

Nine years ago, Paige purchased 500 shares of Earthlink, Inc., for $15,000. Paige recently gifted those shares of stock to her daughter, Phoebe. The value of the 500 shares of stock on the date of the gift was $5,000. Which of the following statements is correct? A) If Phoebe subsequently sells the shares of Earthlink, Inc., for $4,000, the basis used to calculate her gain or loss will be $15,000. B) If Phoebe subsequently sells the shares of Earthlink, Inc., for $20,000, the basis used to calculate her gain or loss will be $5,000. C) If Phoebe subsequently sells the shares of Earthlink, Inc., for $9,000, the basis used to calculate her gain or loss will be $15,000. D) If Phoebe subsequently sells the shares of Earthlink, Inc., for $12,000, she will not have any gain or loss.

D) If Phoebe subsequently sells the shares of Earthlink, Inc., for $12,000, she will not have any gain or loss. The correct answer is (D).Phoebe will have a double (dual) basis in the stock, determined as follows: If Phoebe subsequently sells the shares of Earthlink, Inc., for $12,000 (between the original basis and the fair market value), no gain or loss will be recognized. LO 10.2

All the following taxpayers would be ineligible to claim the student loan interest deduction, EXCEPT A) John, who is the primary borrower on his Federal Stafford loan and claimed as a dependent by his parents. B) Jill, who made payments on her dependent son's Federal Stafford loan. C) Julie, whose employer made all of her student loan payments this year under an IRC 127 plan. D) Jennifer, who made payments on her loan and is not claimed as a dependent.

D) Jennifer, who made payments on her loan and is not claimed as a dependent. The correct answer is (D).Only the primary borrower is eligible to claim the student loan interest deduction, assuming they are also not claimed as a dependent of another taxpayer. As a result of the CARES Act, student loan interest payments made by an employer cannot also be used by the employee to claim the student loan interest deduction (no double dipping allowed). Jennifer is eligible to claim the student loan interest deduction because she is the primary borrower and is not claimed as a dependent of another taxpayer. LO 6.3

Leo's son, Lucas, completed his freshman year of college in 2021. Leo paid $5,000 in qualified expenses for Lucas this year. Leo is a married-filing-jointly taxpayer and has an adjusted gross income (AGI) of $110,000 for the current year. What education credit will provide Leo with the greatest tax savings, and how much is that credit? A) Leo can claim a lifetime learning credit in the amount of $1,000. B) Leo can claim a lifetime learning credit in the amount of $5,000. C) Leo can claim an American opportunity tax credit in the amount of $5,000. D) Leo can claim an American opportunity tax credit in the amount of $2,500.

D) Leo can claim an American opportunity tax credit in the amount of $2,500. The correct answer is (D).Even though Leo can claim the lifetime learning tax credit in the amount of $1,000 (or $5,000 × 20%), the American opportunity tax credit will provide the greatest tax savings in the form of a $2,500 tax credit.($2,000 × 100%) + ($2,000 × 25%) = $2,500 LO 9.3

Which of the following charitable contributions would provide the greatest tax savings for an unmarried taxpayer who is unable to itemize and has an AGI of $10,000? A) A contribution of short-term capital-gain property valued at $7,500 to a private charity B) A contribution of long-term capital-gain property valued at $9,000 to a private charity C) A contribution of real property valued at $8,000 to a public charity D) This taxpayer would not be entitled to a charitable deduction.

D) This taxpayer would not be entitled to a charitable deduction. The correct answer is (D).Since the taxpayer is unable to itemize, only the cash contribution may be claimed as an above-the-line deduction and would be available for tax year 2021. Note, however, that this one-time above the line charitable deduction was eliminated for tax years 2022 and beyond. Since our taxpayer did not itemize his/her deductions, no charitable deduction is available for them. LO 7.3


Kaugnay na mga set ng pag-aaral

Nutrition Exam 1 Practice Questions

View Set

Chapter 5-ENTR-202-Quiz 5 términos

View Set

Property ownership and land use controls and regulations

View Set

HESI Module #6 Safety and Infection Control

View Set